Hyperbolic triangles proof help?

Click For Summary
In hyperbolic geometry, the problem involves proving the relationship between angles θ and ∅ at midpoints M and N of segments AB and AC, respectively. The internal angle sum of a triangle in hyperbolic geometry is less than 180 degrees, which is crucial for the proof. The user expresses uncertainty about how to start, suggesting a need for congruent triangles in the approach. The discussion emphasizes the importance of understanding triangle defects and the properties of midpoints in hyperbolic space. Overall, the focus is on determining whether θ equals, is less than, or is greater than ∅ through a geometric proof.
bonfire09
Messages
247
Reaction score
0

Homework Statement



Prove in hyperbolic geometry: In the accompanying figure M and N are the respective (hyperbolic) midpoints of AB and AC and θ and ∅ are the indicated angle measures. Determine, with proof, which of the following is true:
(1): θ=∅ (2): θ<∅ (3): θ>∅ ( stands for phi)


Homework Equations


I have attached a picture of the figure too. Since I am in hyperbolic geometry I get to assume that the internal angle sum of a triangle <180 degrees and AAA congruence for triangles.


The Attempt at a Solution


I don't know where to get started. It looks simple but I am thinking that I need to get congruent triangles in there somehow but I don't know how to. If you can give me a hint that be great.
 

Attachments

  • triangle.jpg
    triangle.jpg
    3.3 KB · Views: 452
Physics news on Phys.org
bonfire09 said:

Homework Statement



Prove in hyperbolic geometry: In the accompanying figure M and N are the respective (hyperbolic) midpoints of AB and AC and θ and ∅ are the indicated angle measures. Determine, with proof, which of the following is true:
(1): θ=∅ (2): θ<∅ (3): θ>∅


Homework Equations


I have attached a picture of the figure too. Since I am in hyperbolic geometry I get to assume that the internal angle sum of a triangle <180 degrees and AAA congruence for triangles.


The Attempt at a Solution


I don't know where to get started. It looks simple but I am thinking that I need to get congruent triangles in there somehow but I don't know how to. If you can give me a hint that be great.
Where's the figure?
 
I've attached it.
 
bonfire09 said:
I've attached it.
OK I see it now.

I guess that it can take a while to appear initially.
 
The only things that I can really with is triangle defect and the the internal angle sum of triangles are less than 180 degrees. In case of quad's they are less that 360. But I am also given that M and N are midpoints and I know somehow they allow me to do something but not quite sure.
 
Question: A clock's minute hand has length 4 and its hour hand has length 3. What is the distance between the tips at the moment when it is increasing most rapidly?(Putnam Exam Question) Answer: Making assumption that both the hands moves at constant angular velocities, the answer is ## \sqrt{7} .## But don't you think this assumption is somewhat doubtful and wrong?

Similar threads

  • · Replies 1 ·
Replies
1
Views
2K
  • · Replies 3 ·
Replies
3
Views
3K
  • · Replies 1 ·
Replies
1
Views
3K
  • · Replies 9 ·
Replies
9
Views
2K
Replies
5
Views
2K
  • · Replies 24 ·
Replies
24
Views
2K
  • · Replies 1 ·
Replies
1
Views
1K
  • · Replies 14 ·
Replies
14
Views
4K
  • · Replies 7 ·
Replies
7
Views
4K
  • · Replies 14 ·
Replies
14
Views
2K